Finding the coefficients of a polynomial given some restriction

Click For Summary
The discussion focuses on finding the coefficients a, b, and c of the polynomial az^3 + z^2 + bz + c, given the condition that the zeros satisfy the equation z_1^3 + z_2^3 + z_3^3 = 3z_1z_2z_3. Participants reference Vieta's formulas to express relationships among the roots, specifically z_1 + z_2 + z_3 = -1/a, z_1z_2 + z_1z_3 + z_2z_3 = b/a, and z_1z_2z_3 = -c/a. There is a suggestion to utilize the expression for z_i^3 to derive a sum of squares of the roots. The conversation highlights the challenge of starting the problem and the need for clearer visual aids.
MatejNeumann
Messages
1
Reaction score
0

Homework Statement


Find all ##a,b,c\in\mathbb{R}## for which the zeros of the polynomial ##az^3+z^2+bz+c=0## are in this relation $$z_1^3+z_2^3+z_3^3=3z_1z_2z_3$$

Homework Equations


we know that if we have a polynomial of degree 3 the zeroes have relation in this case
##z_1+z_2+z_3=-1/a##
##z_1z_2+z_1z_3+z_2z_3=b/a ##
##z_1z_2z_3=-c/a##

The Attempt at a Solution


I've tried doing something with the vietto rules but I have not really gotten anything. Any tip on how to even start the problem would be really appreciated
b0cb62256bf80e1babd12ebc716ce9a9f6bb641c.png
be7630baf32627b0927dfb9d6ca49d3e409057a0.png
 

Attachments

  • b0cb62256bf80e1babd12ebc716ce9a9f6bb641c.png
    b0cb62256bf80e1babd12ebc716ce9a9f6bb641c.png
    8.3 KB · Views: 628
  • be7630baf32627b0927dfb9d6ca49d3e409057a0.png
    be7630baf32627b0927dfb9d6ca49d3e409057a0.png
    7.7 KB · Views: 578
Physics news on Phys.org
There is another symmetry. Say ##z_1\in \mathbb{R}##, then ##z_{2,3}=x \pm iy##.
 
The images in post #1 are unreadable, at least by me.
 
MatejNeumann said:

Homework Statement


Find all ##a,b,c\in\mathbb{R}## for which the zeros of the polynomial ##az^3+z^2+bz+c=0## are in this relation $$z_1^3+z_2^3+z_3^3=3z_1z_2z_3$$

Homework Equations


we know that if we have a polynomial of degree 3 the zeroes have relation in this case
##z_1+z_2+z_3=-1/a##
##z_1z_2+z_1z_3+z_2z_3=b/a ##
##z_1z_2z_3=-c/a##

The Attempt at a Solution


I've tried doing something with the vietto rules but I have not really gotten anything. Any tip on how to even start the problem would be really appreciated
View attachment 232485 View attachment 232486

If ##z_i## is a root then $$z_i^3 = -\frac{c}{a} - \frac{b}{a} z_i - \frac{1}{a} z_i^2,$$
and we can find ##\sum z_i^2## from the expansion of ##(z_1+z_2+z_3)^2.##
 
Question: A clock's minute hand has length 4 and its hour hand has length 3. What is the distance between the tips at the moment when it is increasing most rapidly?(Putnam Exam Question) Answer: Making assumption that both the hands moves at constant angular velocities, the answer is ## \sqrt{7} .## But don't you think this assumption is somewhat doubtful and wrong?

Similar threads

Replies
14
Views
3K
  • · Replies 3 ·
Replies
3
Views
3K
  • · Replies 1 ·
Replies
1
Views
1K
Replies
3
Views
1K
Replies
3
Views
2K
  • · Replies 1 ·
Replies
1
Views
2K
  • · Replies 2 ·
Replies
2
Views
2K
  • · Replies 4 ·
Replies
4
Views
2K
Replies
4
Views
3K
  • · Replies 5 ·
Replies
5
Views
2K